Sie sind auf Seite 1von 19

1 TEM4SEK JUNCR COLLEGE, SINGAPORE

It is given that u\ 2 and u *\ u, = 2(r + 1) where r e Z*


r

>0 [5]

By using the method of difference, find u in terms of n.


n

fUl
II - Prelimirtary Examination 2011 Higher 2 2 Given that a is a positive constant, solve the inequa! ty \x-2a\<2x
;

+a

[4] [2]

Hence solve the inequality \x + 4| < 2 - 2x.

MATHEMATICS Paper 1

9740/01
Functions f and g are defined by 15 September 2011 tixt+ljl-x* g:*-*lnx, (i) (ii) , xeR,-l<x<l, R * .

Additional Materials:

Answer paper List of Formulae (MF15)

3 hours

Show that composite function gf exists and find the range of gf in exact form. [4] Describe the transformations such that the graph of y = g(x) is mapped onto the graph ofy = l n ( 2 x - 1 ) . [2]

READ THESE INSTRUCTIONS FIRST Write your Civics group and name on all the work that you hand In. Write In dark blue or black pen on both sides of the paper. You may use a soft pencil for any diagrams or graphs. Do not use staples, paper clips, highlighters, glue or correction fluid. Answer all the questions. Give non-exact numerical answers correct to 3 significant figures, or 1 decimal place In the case of angles in degrees, unless a different level of accuracy is specified In the question. You are expected to use a graphic calculator. Unsupported answers from a graphic calculator are allowed unless a question specifically states otherwise. Where unsupported answers from a graphic calculator are not allowed In a question, you are required to present the mathematical steps using mathematical notations and not calculator commands. You are reminded of the need for clear presentation In your answers. The number of marks is given in brackets [ ] at the end of each question or part question. At the end of the examination, fasten all your work securely together. It is given that S. (I) (ii) (iii) (iv) --. tjr(r-l)
2

Write down the values of S .

and S*.

[1] [1] [4] [1]

Make a conjecture for S in terms of n, for n e Z", n 2. Prove your conjecture by the method of induction. Determine if the series J\T-r t$r{r-\) converges.

A graphic calculator is not to be used in answering this question. Two complex numbers z and w are such that z ^3 + i and (i) the fourth roots of z, I = 2i. Find [4] [4]

(ii) the modulus and argument of w.

A curve has parametric equations given by x = 2 0 - s i n 2 0 and y = 2 - c o s 2 0 . This document consists of 4 printed pages. (i) Show that ^ = cot0. dx

[31 [3]

T E M A S E X JlMOFtOOtlESE. SNGAKME l&21&ul [Turn over

(ii) Find the equation of the normal to the curve at the point where 8 - . 4 (iii) For the part of the curve where 0 < 6 < 2K , find the coordinates of the point where the tangent is parallel to the y-axis.

[3]

TJC/MA 9740/Prellmlnarv Exam 2011

Given that y - In (cosx), show that f -1 Ax*

. \dxj

[2]

11

(a)

The graph of y - ft*) is shown below. The curve cuts the y-axis at

^~^j
0

811(1

Determine the first two non-zero terms in the Maclaurin series for y. Use this series to find an approximation for In 2, in terms of n.

[4] [3]

the ;t-axis at (1, 0) and (7, 0). Sketch the graph of y = ~ . showing clearly the I (X) main relevant features of the curve. . [3]

The vector equations of lines /i and h are given by r - 71 + 3j+ 7k + M.I + k), and r - 3j+ 3k + p(qi + 2k), respectively, where A, fl and q e R . It is given that l passes through the points A and B with position vectors 3j+ 3k and gl+3j+5k respectively.
2

(I) (Ii)

If the acute angle between l\d / is 6 0 , find the exact values of q.


2

[3]

If < 7 = 1, find the position vector of the point C on /| such that A is the foot of perpendicular from C to /j. [4] (b) The curve C has equation y
X + P

(ill)

Let 7 * be any point or (|, If the area of triangle ABT is constant for any X, find the value of q. [2]

- , wherep is a non-zero constant.

*(* + 3) (i) (a) Find the sum of all the integers between 1102 and 2011 (inclusive) which are not divisible by 3. [5] A geometric progression G has first term a and common ratio r. A sequence H is formed by squaring the terms of G. Prove that H is also a geometric progression. The sum to infinity of G is S. Find the value of r such that the sum to infinity of His2S . [4]
2

State the equations of the asymptotes.

[2] [4]

(ii) Show that if C has 2 stationary points, then p < 0 or p > 3.

(b)

(iii) Given p = 4, sketch the curve C, showing clearly the equations of the asymptotes and the coordinates of the axial intercepts and stationary points. [2]

12

The line / passes through the points (2, - 1 , - 1 ) and (6, 1, 3). The equation of the plane p is given by r (i + 2j + 2k) - 6.
t

(i) 10 (a) On a diagram, shade the region bounded by the curve y and the lines y 2x x and y 3x for x 2 0 . Label all the intersection points clearly. Find the exact area of the shaded region. (b)
J J

Write down a vector equation of / . Show that the position vector of the point of intersection of / and pi is 41 + k. [3]

(ii)

[2] [3]

A plane p2 contains / and meets the plane p\n a line m . Given that the line m is perpendicular to / , show that m is parallel to 2i + 2j -3k. Hence write down a vector equation of pi.
3

[2] [1 ]

Find the volume of the solid formed by rotating the circle ;c + y - 4 y + 3 = 0 about the x-axis. Give your answer correct to one decimal place. [4]

A plane p has equation x - 7 y - 4 z + it(;c+2y + 2 z - 6 ) = 0. Explain why m lies on pi for any constant k. Hence or otherwise, find a cartesian equation of the plane, in which both m and the point (0, 0, 2) lie. [4]

[Turn over so so so End of Paper ca eg cs

TJC/MA 97/Prellmlnarv Exam 2011

TJC/MA 9740/Prellmlnary Exam 2011

2
ry.

Section A: Pure Mathematics [40 marks] TEMASEK J L N C R COLLEGE, SINGAPORE II . J = _ Preliminary Examination 2011 Higher 2 1 The nth term of a sequence is denoted by a , . The sequence is defined by
a

.i

P . <7 for all n 1, where p and q are constants.


a + 2 i

The first three terms of the sequence are given by a, = 0, a 2 and a = 7 . (i) Find the values of p and q. Express a, in terms of n, for n > 3. [2] [4]

MATHEMATICS
Paper 2

9740/02
19 September 2011

(ii)

Additional Materials:

Answer paper Graph paper List of Formulae (MF15)

3 hours

(a)

Given f ( * )

, identify which of the following graphs, A, B and C is

that of y f(x). justifying your answer clearly. i READ THESE INSTRUCTIONS FIRST Write your Civics group and name on all the work that you hand in. Write In dark blue or black pen on both sides of the paper. You may use a soft pencil for any diagrams or graphs. Do not use staples, paper clips, highlighters, glue or correction fluid. Answer all the questions. Give non-exact numerical answers correct to 3 significant figures, or 1 decimal place In the case of angles in degrees, unless a different level of accuracy is specified in the question. You are expected to use a graphic calculator. Unsupported answers from a graphic calculator are allowed unless a question specifically states otherwise. Where unsupported answers from a graphic calculator are not allowed In a question, you are required to present the mathematical steps using mathematical notations and not calculator commands. You are reminded of the need for clear presentation In your answers! The number of marks is given in brackets [ ] at the end of each question or part question. At the end of the examination, fasten all your work securely together. 4 Given that a = 1 + i , and the complex numbers z and w are such that | z - a | | a | and a r g ^ ^ j - ^ j i - a r g a . Sketch on a single Argand diagram the loci of points representing z and w. This document consists of 6 printed pages. Hence find (1)
CTJC2011

[2] c

(b)

A point P(x, y) moves on the curve y = tan x in such a way that the x-coordinate of P increases at a constant rate of 4 units per second. Find the rate of change with respect to time of the gradient of the curve when x = . 4 [4]

(a)

Find J

fr,

[4]

(b)

Using integration by parts, find the exact value of J xjlx


o

+ l dx .

[4]

[5]

the least value of | w + 31, the greatest value of arg(z + 3).

[2] [3]

[Turn over

(ii)

;
TJC/MA 9740/Preliminary Exam 2011

At time I = 0, there are 8000 fish in a lake. At time t days the birth-rate of fish per day is equal to one-fortieth ( ^ ) o f the number N of fish present. Fish are taken from the lake at the rate of 150 per day. dN Modelling W as a continuous variable, show that 4 0 N - 6 0 0 0 . [2] di Solve the differential equation to find N in terms of t. Find the time taken for the population of fish in the lake to increase to 12000. [5]

Nine balls numbered from 1 to 9 are placed into a bag. The odd-numbered balls are red in colour and the even-numbered balls are blue in colour. Three balls are drawn from the bag without replacement. For each ball drawn, the

number on the ball represents the score. For any blue ball drawn, the score on the next ball drawn will be doubled. [For example, if balls numbered 3, 4, 5 are drawn in this order, the total score is3 + 4 + 5 x 2 1 7 ] Find the probability that (i) (ii) all balls are red, there is at least one ball of each colour, the total score is more than 18, given that all balls are the total score is even. red, [2] [2] [3] [3]

When the population of fish has reached 12000, the number of fish taken from the lake is increased from 150 per day to r per day. Write down, in terms of r, the new differential equation satisfied by yV. [1]

(iii) (iv)

Find the maximum value of r *o that the population of fish in the lake will not decrease. [2]
Section B^Statistics [60 m a r k s ]

9 (a) How many different nine-digit numbers can be formed from five "0"s and four "1 "s such that no three "0"s are together? (b) [4]

(a)

A junior college intends to make changes to its school uniform and decides to obtain the views from students on the design of a new uniform. Mr Tong obtains a sample by selecting 50 students randomly from the school register. (i) Name the sampling method used. [1]

Six people sit at a round table with eight identical chairs. Find the number of seating arrangements if (I) (II) there are no restrictions, the seats are numbered. [2] [1]

(11) Comment on the appropriateness of the use of this method for this situation.

m
(iii) Describe briefly another sampling method which can be used by the school and state one advantage of the method used in this context. [3]

The heights of a randomly chosen boy and a randomly chosen girl in a junior college are denoted by B and G respectively. The random variables B and G are independent normal variables with means and standard deviations as shown in the table. Mean (cm) Boys (B) Girls (G) (i) 170 155 Standard deviation (cm) 4 5

(b)

A machine is set to fill .jars with 270 g of kaya. Over a period of time it has been established that the standard deviation of the mass of kaya in a jar is 0.8 g. A setting on the filling machine is altered and the operator suspects that the mean mass of kaya filled by the machine has decreased. A random sample of 10 jars of kaya were measured and the masses in grams were found to be 269, (1) 271, 270," 268, 269. 270. 268, 270. 271, 269.

Explain briefly why the assumption of a normal distribution with the above mean and standard deviation for the heights of the boys may be reasonable even though such a height cannot possibly be negative. [1]

Assuming that the standard deviation may have been altered, perform an appropriate test at the 10% level of significance, on whether the operator's suspicion about the machine is justifiable. State any assumptions make in carrying out the test. [5]

(ii)

Find the probability that 3 times the height of a randomly chosen girl will exceed the total height of 2 randomly chosen boys by more than 120 cm. [3] (ii)

Assume that the standard deviation of the mass of kaya in a jar is unaltered. State what test you would use in this case, giving any assumptions that you need to make. [You are not required to carry out this test]. [1]

(iii) (iv)

Find the value of k such that P(|G -155| < k) = 0.85

[3]
[2] [Turn over

Three boys are chosen at random. Find the probability that the first chosen boy is taller than 165 cm and the other two boys are shorter than 165 cm.

TJC/MA 9740/Prellminary Exam 2011

TJC/MA 9740/Prelimlnary Exam 2011

00 10 (a) The scatter diagram shows a sample of size 6 of bivariate data, together with the regression line of yonx. 11 (a) X is a discrete random variable with mean 20.5 and variance 3.2. A random sample of 60 observations of X is taken, find th- probability that the sample mean lies between 19 and 20. (b) [3]

Analysis of the scores in football matches in a local league suggests that the number of goals scored in a randomly chosen match may be modelled by the Poisson distribution with mean 2.7. State, in the context of the question, one [1] assumption for the Poisson distribution to be a valid model.

State and giving a reason, whether, for the data shown, the regression line of y on x is the same as the regression line of x on y. [1] (I) Find the probability that a randomly chosen match will end with at least two goals scored. (ii) [2] [1]

Given that y = -8x+86 is the regression line of y on x for the 6 pairs of observations with V x = 60. (i) (ii) Find the value of the mean of y. State, with a reason, the regression line of y on x after a new pair of observation (10,6) is added. (b) [2]

On one afternoon, 12 matches are played in the league. Find the probability that there are fewer than 8 matches in which there are at least 2 goals scored. [2]

T is the total number of goals scored in n matches. State the distribution of T and its parameters). [1]

A car is travelling along a stretch of road with speed v (km/h) when the brakes are applied. The car comes to rest after'travelling a further distance of d m. The values of d for 10 different values of v are given in the table, correct to 2 decimal places. V 35 4.90 40 5.30 45 5.93 50 6.45 55 7.00 60 8.50 65 12.00 70 75 80

Using a suitable approximation, find the least value of n such that the probability of T being greater than 100 is more than 0.9. [4]

17.00 22.10 25.90 BO so BO E n d of Paper w w w

It is given that the value of the linear product moment correlation coefficient for this data is 0.919, correct to 3 decimal places. The scatter diagram for the data is shown below.

(1)

Calculate the product moment correlation coefficient between v and 4d . What does this indicate about the scatter diagram of the points (v, -Jd )? [2]

(ii)

Which regression line is more appropriate: 4d

on v or d on v? State the [2]

reason and find the equation of the appropriate regression line. (iii)

Estimate the distance a car travels after its brake is applied when the car's speed is lOOkm/h. Comment on the reliability of your answer.
TJC/MA 9740/Preliminary Exam 2011

[2] [Turn over


TJC/MA 9740/Preliminary Exam 2011

xuuu 1 Method of Difference

2011 Preliminary Exam H2 Maths 9740 Paper 1 Solutions

3 (D

Functions & Transformations

(*-*)-

*Rr+n

>--f(x)

Since R - ( 0 , 2] c R * - D , , gf exists.
f

Method 1 (mapping method) (-1, 1) => u . - u , = ( - l X " + 2)


M.

(0,2] - L . . ( - , In 2]-Range of gf

Thus

n - 2 + 2 = n(n + l) u.*n(n + l)

Method 2 (graphical method) gfl&r)- l n ( 2 v ' l - x ) - b 2 + ^ l n ( l - x ) , - l < x < J


J ,

[Alternative M e t h y l Summing up to n terms, we obtain u M, /(n + 3) = n + 3 / i


mti J

, - n * n + 2 ( n + l)(n + 2)
1

y - gfl

Thus

M=(n + 1)

Inequalities Range of gf-(-oo, In 2] (ii) A translation of 1 unit along the positive x-axis followed by a scaling of factor ^ along the x-axis. OR A scaling of factor along the x-axis followed by a translation of unit

along the positive x-axis, To find intersection point, 2x + a = -(x - 2a) x 3 From the graph, for |x - 2a| < 2x + a, x>a 3

Replace x by -x and let a - 2 in the above inequality, |(-x) - 2(2)| < 2(-x) + 2 becomes |x + 4| < 2 - 2x ^ . 2 2 Thus -x>-=>x<-TJC/MA8740/P1/Preliminary E x a m 2011/SohJtlons TJC/MAB740/P1/Prellmlnary E x a m 2 0 1 1 / S o h J t a n i

(0

Mathematical Induction I l " 2(1) * 2


m 1

Complex Numbers (0 |*|>/3772 argz * *-tan"

5, + ' 2 3(2) 3
4 -

4{3)"4

( , , )

t . = i . . 3 r' - 2 * | e ^ ~ ' | , it--2,-1.0,1 Let / \e the statement 5. n When - 2, LHS - 5, = - from (i) .
2

( H , )

where n I Z * , * 2. i
e

i . , 2 , i , 1 . i ,!!. , 2 e , 2 ' e " , 2*t


4

4
tir(r-l)

.-. /i if true Assume Pt be true for some k e Z*, A : 2, i.e. 5, y.-+ ^r(r-l) (* + !)* k-l 1

bi.j

=>-iarg(w)-10arg(2)-|

(*-lX* + Q-l (* + !)* (* + i)* Thus Pt*\s true. Since Pi is true, and P .\s true if Pt is true, by the method of mathematical induction, P is true for all n e Z', n 2 2.
t H

_ . . , . . 47* * ,*, Principal arg(w) - loir m-

*+i

*+i -2i

2e

As n - oo, 5'

1 . Thus >

converges to 1.

= 2e"

t?

2e

w2 e^~-2 e"^
n u

H " *
2

arg(w)-y

TJC/MA9740/P1/Preliminary E x a m 2011/Solutlont

TJDMA970/Pt/Pre<nnlnary E x a m 2011/SoftiDon i

6 (i)

Tangents & Normals (Parametric Equations) = 2-2cos26> , i ^ - 2 s i n 2 0 60 d0 dy 2sin2fl dx 2-2cos20 _ 4sinflcosfl COtf? 4sin f?
2

(I)

Vectors Given acute angle between l\d (1)

is 60*, Apply modulus to ensure an acute angle is obtained regardless of value of q.

cos 60 = 1^2) = - ^ # 7 4
(fl +

At the point where 0= , x = - 1 , 4 2

y = 2,

= cot= 1 dx 4

2) =i(^ 4)
2 +

Equation of the normal: y-2 = -(jr+1) q + Sq + 4 - 0


2

y= l+ ^

--x Marker's comments: The modulus on the scalar product was missing even when the angle was given to be acute in the question. Among answers which applied the modulus, the modulus "disappeared" right after the scalar product had been evaluated to be q + 2. Many students made careless mistakes in expanding 2(q + 4) and (q + 2) .
2 2

When the tangent is parallel to they-axis, is undefined dx For ^ to be undefined, s i n # 0 sine? => 0 = rt for 0 < 0 < 2/r When 0 = n, x = 2/r, y = 1 .'. the tangent is parallel to the y-axis at the point (2rt, 1).
C 0 S

(ii)

'1 Since C lies on /,, &C = 3

m 0 for some X

Maclaurin's Series y - In (cosx) dy -sinx = -tanx dx cosx ^ = - s e c x = -(l + t a n x ) - - 1 2 2

Obtain OC in terms of one unknown X only.

JC=UC-Ua

4+AJ Since AC 1 / AC d = 0 ,
2

B9

,4(0,3,3)

dx
4

dx[dx
2

2y

[1*1 0 -5

fd y dy 'dV^l = -2 ,dx 1 d x dx
4 2 +

.-. o~cWhenx = 0 . y 0 , ^ - 0 , ^ - - 1 , ^ - 0 , dx dx dx
2 3 y

^ - 2 . dx
4

"

2! " 4!

12

(hi)

Since the area of ABT is constant for all k, l\d /; are parallel. V c =* o X k A

Taking x = , 4
In

4' it 32 rt 3072 => in 2 + 16 1536

9=2

1 . _ In2 2

T J C / M A 9 7 4 0 / P 1 /Preliminary E x a m 2011/Solutlon

TJCVMAJV40/P1/Preliminary E x a m

2011/Sowtoni

[Alternative solution] Since flies on l\, UT = 3 T 0 for some X J) Area of MBT =-\

9 (a)

AP&GP Sum of all integers between 1102 and 2011 inclusive 2011-1102,l ^ ,
= ( l l 0 2 2 0 | l ) l 4 1 6 4 1 s

1104,1107,2010 is an AP with first term 1104 and common difference 3 2010-1104 + ( n - l ) 3 => n 3 0 3 0 X A l4-4? + ( 2 - * M 2 (b)

7+yl

Sum of integers between 1102 and 2011 that are divisible by 3 = ^ ( 1 1 0 4 + 2010) = 471771 .. required sum- 1416415-471771 -944644

.|14-4o + (2-aM| Since the area is a constant independent of X, QHSflB .-. - 2 Marker's comments: Quite a number of students left the part unanswered. Some used the wrong formula eg
9

Let (J* and H be the nth terra of G and H respectively. 2

* > constant which is independent of n


a

Therefore, H is a geometric progression.

^ 1 AB || AT\, J AB*AT

|. For students who attempted to use the cross-product formula, l-r


J

many did not bow to proceed after obtaining the area. Many students made mistakes in 0 1 (14 - 4o + (2 - q)X) and evaluating 14-4<7 + (2-<7)A ; some wrong answers were 2

[,1-r,

(.-r)'=2(W) 3r -2r-l =0
J

r or r- 1 (rejected since |rj< 1)

TJC/MA9740/P1/Prellmlnary E x a m 2011/Solutlons

T J C / M A 9 7 4 W P ^Preliminary E x a m 201 P M U t f O M

10 (a)

Applications of Integration (Area & Volume)

11

Graphing Techniques & Graph o f f '(x) fO

(a)

A-I(VJ)(3V5) ^fd,-I(VS)(2V5)
+

-3(ln3-ln2)
->v

: !l

l 3

i 5

-2

(b) (b)(1)

\
X1 x-7

(U)

Horizontal asymptote: y - 0 Vertical asymptotes: x - 0, x - -3 , ( , ' + 3 x ) - ( x + p)(2x + 3) dx"


J

x*(x + 3)*
J

_ (x + 3 x ) - ( 2 x +3x + 2/7x + 3p)


=

x*(x 3)
+ 2

_-x -2/?x-3/> x (x + 3)
J J

x ' + y - 4 y + 3 0 =>x +(y-2) -I


a 1

Volume- 2 ^ ( / r y , - / r y ) d x where y, =2 + V l - x
2 2J 2 2

and y = 2 - V l - * . (iii) dx 39.5 using GC

^ ( x + 3)* = x + 2 / + 3/>0 Since C has two stationary points, discriminant > 0 =*4p -12p>0 p(p-3)>0 p < 0 or p > 3 (shown)
l ,

dx

(-i\-l)

x - - 3

JT-0
10

TJC/MA97407P1/Prelimlnary E x a m 2011/Solutlont

TJCAIA9740/P1/Pfeliminafy E x a m 2 0 J ITSolutnna

12 (0

Vectors 6^ (2 *j A directional vector of / is 1

' 1 - 1 2 1
2

.3; r l r *> '2> Equation of / is r = -1 + a 1 . a c t ,2, -1.


2

(ID

(2\ -1 +a 1 I 2 -2 + 8a = 6 .-.a-I ( 2> Thus the position vector of point of intersection -1


+

4 * 1 '2 1 0 A I'.

(ill) A vector // to m

[21 [1*1 r -2] 1 X 2 -2 .3. .2, ^2

r2 * 1 2 :. m //

Alternative method [1*1 [2*1 '1 { 1 2 2 = 0 and 2 1 -3; ,2, -3 .2,


2

-0

(Iv) is parallel to vectors

[2*1 ^ 1 l and 2 ,2,

Hence, equation of the plane 11, is r' '4> [2*1 Equation of line m is r 0 + 0 2 l-3>

f ]
2

-1

2\ [2*1 l + t* 2 ^-3 v2,


r

Substitutes-4 + 2/?,>-= 2p and z - 1-3/9 into the equation of n , L H S - 4 + 2 9-14 9-4 + 129 + *(4 + 2 5 + 4/? + 2 - 6 5 - 6 ) 0 for any P and k Thus every point on m is on the plane FI, for all k. Hence m lies in n , for all k.
/ / / / /

Since (0, 0, 2) lies on n , , substitutex - 0, y - 0 , r - 2 into the equation of n , -8 + *(4-6) = 0 =>/c--4 .. equation of plane is x - 7y - 4r - 4(x + 2y + Iz - 6) = 0 i.e. x + 5y + 4r - 8
TJC/MA9740/P1/Preliminary E x a m 2011/SoluIlon T J C / M A S 7 4 0 / P 1 ' P r e fcrtr.ary E x a m 2011/ScJullona

Section A:

Pure Mathematics

[ 40 m a r k s |

T E M A S E K JUNIOR C O L L E G E . S I N G A P O R E

The nth term of a sequence is denoted by a,. The sequence is defined by a, pa. + q for all n 2i 1, where p and q are constants. The first three terms of the sequence are given by a, - 0, a - 2 and a - 7.
2 y

J~L

Preliminary Examination 2011 Higher 2

(I)

Find the values of p and q. Express a in terms of n, for n it 3. I -2 7-2p + 2 5

[2] [4]

MATHEMATICS Paper 2
Additional Materials: Answer paper Graph paper L i s t of F o r m u l a e ( M F 1 5 )

9740/02
19 S e p t e m b e r 2011

(II)
[Solution]

3 hours

(0 a -pa +q
t }

R E A D T H E S E INSTRUCTIONS F I R S T

(li)
W r i t e y o u r C i v i c s group a n d n a m e o n all the w o r k that y o u h a n d In. W r i t e in d a r k b l u e or b l a c k p e n o n both s i d e s of the p a p e r . Y o u m a y u s e a soft p e n c i l for a n y d i a g r a m s o r g r a p h s . D o not u s e s t a p l e s , p a p e r c l i p s , highlighters, g l u e or c o r r e c t i o n fluid.

Given a.., = - a + 2, Oj 2 5 a,--a,+2

nil.

A n s w e r a l l the q u e s t i o n s . G i v e n o n - e x a c t n u m e r i c a l a n s w e r s c o r r e c t to 3 s i g n i f i c a n t f i g u r e s , or 1 d e c i m a l p l a c e In the c a s e of a n g l e s in d e g r e e s , u n l e s s a different l e v e l of a c c u r a c y I s s p e c i f i e d In the q u e s t i o n . Y o u a r e e x p e c t e d to u s e a g r a p h i c c a l c u l a t o r . U n s u p p o r t e d a n s w e r s from a g r a p h i c c a l c u l a t o r a r e a l l o w e d u n l e s s a q u e s t i o n s p e c i f i c a l l y s t a t e s otherwise. W h e r e u n s u p p o r t e d a n s w e r s from a g r a p h i c c a l c u l a t o r a r e not a l l o w e d in a q u e s t i o n , y o u a r e r e q u i r e d to p r e s e n t the m a t h e m a t i c a l s t e p s u s i n g m a t h e m a t i c a l notations a n d not c a l c u l a t o r c o m m a n d s Y o u a r e r e m i n d e d of the n e e d for c l e a r p r e s e n t a t i o n In your a n s w e r s . T h e n u m b e r of m a r k s Is g i v e n in b r a c k e t s [ ) at the e n d of e a c h q u e s t i o n or part q u e s t i o n . A t the e n d of the e x a m i n a t i o n , f a s t e n all your w o r k s e c u r e l y t o g e t h e r .

-f (2)

+2
+

* , - 0 2 )

2) 2
+

"(fT *f
(2)

(2)+2

-2 1 +

T h i s d o c u m e n t c o n s i s t s of 6 printed p a g e s .

m i
Vti&tiNk

JUMK

course, SKGATOW
warn M * G M
CMM

[Turn over

-iter-

for

n 21 '>

(a)

Given f \x) =

. identify which of the following graphs. A, B and C is [2]

(a)

Find f
1

, * dx. x -2x + 3

[4] [4]

that ofy f(x), justifying your answer clearly.

(b) (Solution]

Using integration by parts, find the exact value of J xV2x +1 dx

x -2x + 3
J

x -2x + 3 1r 2x-2 r 1 . = - : dx + : dx 2 x"-2x + 3 x *-2x + 3


1 3 J J 1

(Ml]

= -ln|x"-2x+3| + f (b) A point P{x, y) moves on the curve y tan x in such a way that the x-coordinate of P increases at a constant rate of 4 units per second. Find the rate of change with respect to time of the gradient of the curve when x - . (Solution] () Given f ( x ) l + 1 2 J

rdx

[4]

( - . ^ ( ^ 1. . j . .. 1 .ifx-0 = - l n x -2x + 3 + -wtan 7 - - +c 2 ' 72 IV2 J

( * ) Since f (0) I but the gradient of the graph C at x 0 is undefined, graph C cannot be the graph ofy - fi.
2

(b)

| W 2 x + l dx

(2x + l)

Since f (x) * 0 for all real x, y - f(x) has no turning points. Therefore graph A cannot be the graph ofy - f(x). (Or) When x -* oo, f '(*) 0. Graph A cannot be the graph ofy - tTx).

= 1(373-0) - 1

(2x + l)

2>/3

Hence, Graph B is the graph ofy - f(x). (b) y t a n x = = sec x. dx


2

Given that 4 . Let G - d/ dx dG dC dx dr dx d/ -(2sec xt.nx)x^


J

correct , apply chain rule. d|

Whenx-^,

^. (72) (.)(4)-16
2 2

Given that a = I + i , and the complex numbers z and > v are such that | r - o | = | a | and arg^-^-^-j = arga. Sketch on a single Argand diagram the loci of points representing z and M . Hence find (I) (II) the least value of | w + 31, the greatest value of arg(z + 3). [2] [3] [5]

At time / - 0, there are 8000 fish in a lake. At time / days the birth-rate offish per day is equal to one-fortieth ( ^ ) o f the number /V of fish present. Fish are taken from the lake at the rate of 150 per day. Modelling N as a continuous variable, show that 4 0 = N - 6000. dt [2]

Solve the differential equation to find /V in terms of /. Find the time taken for the population of fish in the lake to increase to 12000. [5] When the population of fish has reached 12000, the number of fish taken from the lake is increased from 150 per day to r per day. Write down, in terms of r, the new differential equatipn satisfied by N. [1]

[Solution] a - l + i , l a l - V l + l "72 w-2\>r org 2i J 4 arg(w-2) = - + arg(2i) 4


. it
+

and

arga = 4

Find the maximum value of r so that the population of fish in the lake will not decrease. [2] [Solution) Birth-rate of fish per day - N ^ 40 Rate of fish removed from the lake per day - 150
7

It

lit
T

=>arg(w-2)--

- -

= /V-150 => 4 0 - / V - 6 0 0 0 d/ 40 dt 40 f
J

1 oW-fldf A'-6000 J

40ln|tf-6000|-/ + c | iV-6000 | - e*> N- 6000 + Ae , where


i

A-te"

When / - 0, N - 8000, A - 2000 .-. N -6000 + 2000 e *


3

When N - 12000, t - 40 ln3 - 43.9 days (1) In MBF, \AF] - 5, ZBFA = n/4. n| j ] " ^ ^ (11) Greatest value of arg(z + 3) - ZDAE =ZDAC+ZCAE .06+14.04-34.1 When N - 12000, number of fish taken from the lake - r per day. New di fferential equation is /V - r dt 40 d/V For population of fish in the lake not to be decrease, 2: 0. di When N - 12000, - (12000) - r - 300 -/ 2:0 dt 40 =rS300 The maximum value of r is 300. |

+un

UJ

Section B: Statistics |60 marks] (a) How many different nine-digit numbers can be formed from five "0"s and four " 1 "s such that no three "0"s are together? [4] Six people sit at a round table with eight identical chairs. Find the number of seating arrangements if (I) there arc no restrictions, [2]
[1]

The heights of a randomly chosen boy and a randomly chosen girl in a junior college are denoted by B and G respectively. The random variables B and G are independent normal variables with means and standard deviations as shown in the table. Mean (cm) Standard deviation (cm) 4 5

(b)

Boys (B) Girls (G)

170 155

(II) the seats are numbered. [Solution| (a) Case 1: All "0"s separated. Case 2: One "00" , three "0"s separated. Case 3: Two "00"s, one "0", Therefore total number of ways = 51.

(I) Explain briefly why the assumption of a normal distribution with the above mean and standard deviation for the heights of the boys may be reasonable even though such a height cannot possibly be negative. [1] (II) Find the probability that 3 times the height of a randomly chosen girl will exceed the total height of 2 randomly chosen boys by more than 120 cm. [3] (Ui) Find the value of* such that P(|G-155|<*)0.85 [3]

1 way. 2 0 ways 3! 3!
7

2!

30 ways.

Alternative method: ;$j-S-A' (00000)]

(0000. 0) "126-5-20-20-30-51

(lv) Three boys are chosen at random. Find the probability that the first chosen boy is taller than 165 cm and the other two boys are shorter than 165 cm. [-2] [Solution) (I) It is reasonable because P(Height of a boy < 0) * 0 which is negligible.

^(000.00) ] ^ ( 0 O 0 . 0 . 0 ) ]

(11) G-N(155,5 ),
J

B~N(I70,4').
1 J J

E ( 3 G - f l , - , ) 3 x 1 5 5 - 2 x 1 7 0 - 1 2 5 , V a r ( 3 G - 5 , - B , ) - 3 x 5 + 2 x 4 -257 3G - 5, - fl ~ N(l 25,257)


2

(b)

(8-0! (I) Number of ways = ~ ^ - 2520.


L L

P(3G-B,-5,>120)-0.622

(II) Number of ways -

(8)-20160.

(ill)

0-/^(155,25) P(|G-155|<*) = 0.85 => P ( - * < G - 1 5 5 < * ) = 0.85 => P ^ - j < Z < j j - 0 . 8 5 => P ^ Z < - j j = 0.075 . . . . i = _|.4395 = * - 7.20 (OR) P(|G-155|<*) = 0.85 = > P ( - * < G - 1 5 5 < * ) = 0.85 =>P(G<!55-*)-0.075 =5155-*-147.802 => * = 7.20

(lv)

Required probability - P ( B > 165)x[p(fl < 165)] = 0.00998

9 Nine balls numbered from 1 to 9 are placed into a bag. The odd-numbered balls are red in colour and the even-numbered balls are blue in colour. Three balls are drawn from the bag without replacement. For each ball drawn, the number on the ball represents the score. For any blue ball drawn, the score on the next ball drawn will be doubled. [For example, i f balls numbered 3, 4, 5 are drawn in this order, the total score is 3 + 4 + 5 x 2 - 1 7 ] Find the probability that (I) all balls are red, (II) there is at least one ball of each colour, (Ui) the total score is more than 18, given mat all balls are red, (lv) the total score is even.
[Solution!

(lv)

P (Total score is even) = P((E,E,E).(E,E.O).(E,0, E ) , ( 0 , O . E ) ) _ (4x3x2) + (4x3x5) + (4x5x3) + (5x4x4) 9x8x7 284 ~63~9

[2] [2] [3] [3]

Alternative method: P (Total score is even) - 1 - P( ( 0 , 0 , 0 ) , (O, E, E), (O, E, O), (E, O, O)) J '(5x4x3)+(5x4x3) + (5x4x4) + (4x5x4) |
{ >

4 1*9

9x8x7

o r

- x i x - = -0.119 9 8 7 42

(II)

P (at least one ball of each colour) - 1 - P( no R) - P( no B)

I I .3 Alternative method:

' U.
9

P (at least one ball of each colour) - P( {2R1B} or {1R2B}) .2Al) UA2J
+

5
6

0.833

C)
(111) 4,-0.2 S\3
Z

P (Total score > 18 | all balls are red) - P ({5,7,9), {3,7,9} | all balls are red) (OR) 5 ( "
i

" H

' . ! . ,

Alternative method: Ix3!+lx3! _ P( Total score>18 and all balls are red ) P( {5,7,9}, {3,7,9} ) P(all balls are red) " P(aU balls are red) "
m g a 0 2

_5_ 42

11

12

(a)

A junior college intends to make changes to its school uniform and decides to obtain the views from students on the design of a new uniform. Mr Tong obtains a sample by selecting 50 students randomly from the school register. (i) (II) Name the sampling method used. [l]

(b)

(0

From GC, x - 2 6 9 . 5 . J-1.08012


270 / / , : / / < 270

Comment on the appropriateness of the use of this method for this situation. [I] (III) Describe briefly another sampling method which can be used by the school and state one advantage of the method used in this context. [3] (b) A machine is set to fill jars with 270 g of kaya. Over a period of time it has been established that the standard deviation of the mass of kaya in ajar is 0.8 g. A setting on the filling machine is altered and the operator suspects that the mean mass of kaya filled by the machine has decreased. A random sample of 10 jars of kaya were measured and the masses in grams were found to be
269, 271, 270, 268, 269, 270, 268, 270, 271, 269.

Assumption: The mass of kaya in a jar follows a normal distribution since a is unknown and n =10 is small.
1

Test Statistic:

~{L-i.

Level of significance: 10% Assuming Ho is true, using GC, />-value 0.0886 Since p-value - 0.0886 < 0.10, we reject // and conclude that there is sufficient evidence at 10% level of significance that the mean mass of kaya in a jar has decreased.
0

(I)

Assuming that the standard deviation may have been altered, perform an appropriate test at the 10% level of significance, on whether the operator's suspicion about the machine is justifiable. State any assumptions make in carrying out the test. [5] Assume that the standard deviation of the mass of kaya in a jar is unaltered. State what test you would use in this case, giving any assumptions that you need to make. [You are not required to carry out this test]. [1]

(II)

If standard deviation is unaltered, a Z-test is used and we assume that the mass of kaya in a jar to be normally distributed since sample size It 10 is small.

(II)

[Solution] (a) (I) (II) Simple Random Sampling method. Simple Random Sampling is not appropriate as there is a possibility that the sample is not a good representation of the student population. For example, the sample could have an unevenly large number of one gender and less of the other. Stratified sampling could be used where the sample is obtained by dividing the student population according to gender. The students are selected randomly from each gender and the number of students selected from each gender should be proportional to the sizes of each gender in the school. The advantage of this method ensures a good representative sample of the student population as each gender is proportionally represented. Note: Quota Sampling is not acceptable as the samplingframeis available.

(ill)

13

14 (Solution) (a) No, the regression line of y on x is not the same as the regression line of x ony since the points are not collinear. (I) Regression line of y on x: y = -8x + 86,

10

(a)

The scatter diagram shows a sample of size 6 of bivariate data, together with the regression line of y on x.

r-9-io
State and giving a reason, whether, for the data shown, the regression line of y on x is the same as the regression line of x ony. [1] Given that y - -8x + 86 is the regression line of y on x for the 6 pairs of observations with x 60. (I) (II) Find the value of the mean ofy. State, with a reason, the regression line of y on x after a new pair of observation (10, 6) is added. (b) [2] (II) Since ij - - 0.947 is closer to 1 than 0.919, the regression line of Jd on v is more appropriate than the regression line of a on v.
1

(II)

o => y--8(10)+ 86 -6 Since (10, 6) is (x , y ) , the regression line ofy on x remains the same. I

(b) (I) r j . - 0.947. [2] This indicates that the points (v, -Jd ) are lie close to a straight line with positive gradient

A car is travelling along a stretch of road with speed v (km/h) when the brakes are applied. The car comes to rest after travelling a further distance of dm. The values of d for 10 different values of v are given in the table, correct to 2 decimal places. V d 35 4.90 40 5.30 45 5.93 50 6.45 55 7.00 60 8.50 70 75 80 65 12.00 17.00 22.10 25.90

Equation of the regression line of -Jd on v : Jd m

0.53154 + 0.06564v.

i.e.
7

yfd - - 0.532 + 0.0656v.

It is given that the value of the linear product moment correlation coefficient for this data is 0.919, correct to 3 decimal places. The scatter diagram for the data is shown below.

(Hi) Whenv-100, To - -0.53154+ 0.06564(100) = 7.09554 => d~ 50.35 m The estimate is not reliable as it is an extrapolation, v - 100 lies outside the data range.

0)

Calculate the product moment correlation coefficient between v and What does this indicate about the scatter diagram of the points (v, -Jd)? [2] Which regression line is more appropriate: -Jd on v or d on v? State the reason and find the equation of the appropriate regression line. [2] Estimate the distance a car travels after its brake is applied when the car's speed is 1 OOkm/h. Comment on the reliability of your answer. (2]

(ID

(ill)

15

16

11

(a)

A" is a discrete random variable with mean 20.5 and variance 3.2. A random sample of 60 observations of X is taken, find the probability that the sample mean lies between 19 and 20. [3] Analysis of the scores in football matches in a local league suggests that the number of goals scored in a randomly chosen match may be modelled by the Poisson distribution with mean 2.7. State, in the context of the question, one assumption for the Poisson distribution to be a valid model. [1] (I) (II) Find the probability that a randomly chosen match will end with at least two goals scored. [1] On one afternoon, 12 matches are played in the league. Find the probability that there are fewer than 8 matches in which there are at least 2 goals scored. [2]

Alternative solution: Since n is large, by Central Limit Theorem T - X, + X + Xj +....+ X. - N(2.7n. 2.7n) approximately For P(7"> 100) > 0.9
2

(b)

when n - 4 2 , P(7/> 100) - 0.89587 when n - 4 3 , P(7/> 100) - 0.93244 Thus the least value of n is 43

Using GC,

w w w

End of Paper eg c* cu

T is the total number of goals scored in n matches. State the distribution of T and its parameters). [1] Using a suitable approximation, find the least value of n such that the probability of T being greater than 100 is more than 0.9. [4] [Solution] (a) Given E(A) - 20.5 and \u{X) - 3.2 Since n - 60 is large, by Central Limit Theorem, X +X .... X ^ 3^2 , 60 60 P (19 <7< 20)-0.0152
m i l m N ( 2 Q } t p p r o x i m a t e v

(b)

Assumption: Each goal is scored independently of one another. OR The mean number of goals scored is proportional to the duration of the match. OR The goals are scored at a constant rate. (I) Let A T be the number of goals scored in a match X P(XZ2)= 1 -P(XZ 1)-0.751 (II) Let Y be the number of matches in which there are at least 2 goals scored r ~B (12, 0.75133960) P ( y < 8 ) - P ( r s 7 ) - 0.155 T-Xi+Xi + Xi+.-.+X, r ~P (2 .7n) Assuming that 2.7n > 10, T~ N(2.7n, 2.7) approximately
0

P(7"> 100 )*P (r> 100.5) > 0.9 Using GC, when n - 42, ?(T > 100.5) - 0.88713 when n- 43, P(f> 100.5) - 0.92616 Thus the least value of n is 43

Das könnte Ihnen auch gefallen